2014-02-09 16:32:39 +01:00
%!TEX root = GeoTopo.tex
2014-01-17 11:22:58 +01:00
\chapter * { Lösungen der Übungsaufgaben\markboth { Lösungen der Übungsaufgaben} { Lösungen der Übungsaufgaben} }
2013-11-05 11:11:07 +01:00
\addcontentsline { toc} { chapter} { Lösungen der Übungsaufgaben}
2013-11-13 08:48:55 +01:00
\begin { solution} [\ref { ub1:aufg1} ]
2013-11-06 23:03:57 +01:00
\textbf { Teilaufgabe a)} Es gilt:
\begin { enumerate} [label=(\roman * )]
\item $ \emptyset , X \in \fT _ X $ .
\item $ \fT _ X $ ist offensichtlich unter Durchschnitten abgeschlossen,
d.~h. es gilt für alle $ U _ 1 , U _ 2 \in \fT _ X: U _ 1 \cap U _ 2 \in \fT _ X $ .
\item Auch unter beliebigen Vereinigungen ist $ \fT _ X $ abgeschlossen,
d.~h. es gilt für eine beliebige Indexmenge $ I $ und alle
$ U _ i \in \fT _ X $ für alle $ i \in I: \bigcup _ { i \in I } U _ i \in \fT _ X $
\end { enumerate}
Also ist $ ( X, \fT _ X ) $ ein topologischer Raum.
\textbf { Teilaufgabe b)} Wähle $ x = 1 , y = 0 $ . Dann gilt $ x \neq y $
und die einzige Umgebung von $ x $ ist $ X $ . Da $ y = 0 \in X $ können
also $ x $ und $ y $ nicht durch offene Mengen getrennt werden.
$ ( X, \fT _ X ) $ ist also nicht hausdorffsch.
\textbf { Teilaufgabe c)} Nach Bemerkung \ref { Trennungseigenschaft}
sind metrische Räume hausdorffsch. Da $ ( X, \fT _ X ) $ nach (b) nicht
hausdorffsch ist, liefert die Kontraposition der Trennungseigenschaft,
dass $ ( X, \fT _ X ) $ kein metrischer Raum sein kann.
2013-11-05 11:11:07 +01:00
\end { solution}
2013-11-13 08:48:55 +01:00
\begin { solution} [\ref { ub1:aufg4} ]
2013-11-17 21:19:41 +01:00
\textbf { Teilaufgabe a)}
\textbf { Beh.:} $ \forall a \in \mdz : \Set { a } $ ist abgeschlossen.
Sei $ a \in \mdz $ beliebig. Dann gilt:
2014-02-20 16:05:38 +01:00
Wenn jemand diese Aufgabe gemacht hat, bitte die Lösung an info@martin-thoma.de
schicken.%TODO
2013-11-17 21:19:41 +01:00
\textbf { Teilaufgabe b)}
\textbf { Beh.:} $ \Set { - 1 , 1 } $ ist nicht offen
\textbf { Bew.:} durch Widerspruch
Annahme: $ \Set { - 1 , 1 } $ ist offen.
Dann gibt es $ T \subseteq \fB $ , sodass $ \bigcup _ { M \in T } M = \Set { - 1 , 1 } $ .
Aber alle $ U \in \fB $ haben unendlich viele Elemente. Auch endlich
viele Schnitte von Elementen in $ \fB $ haben unendlich viele
Elemente $ \Rightarrow $ keine endliche nicht-leere Menge kann
in dieser Topologie offen sein $ \Rightarrow \Set { - 1 , 1 } $ ist
nicht offen. $ \qed $
2013-11-26 23:00:58 +01:00
\textbf { Teilaufgabe c)}
2013-11-17 21:19:41 +01:00
\textbf { Beh.:} Es gibt unendlich viele Primzahlen.
\textbf { Bew.:} durch Widerspruch
Annahme: Es gibt nur endlich viele Primzahlen $ p \in \mdp $
Dann ist
\[ \mdz \setminus \Set { - 1 , + 1 } \overset { \text { FS d. Arithmetik } } = \bigcup _ { p \in \mdp } U _ { 0 ,p } \]
endlich. Das ist ein Widerspruch zu $ | \mdz | $ ist unendlich und
$ | \Set { - 1 , 1 } | $ ist endlich. $ \qed $
2013-11-05 11:11:07 +01:00
\end { solution}
2013-11-13 08:48:55 +01:00
\begin { solution} [\ref { ub2:aufg4} ]
\begin { enumerate} [label=(\alph * )]
2013-11-26 23:00:58 +01:00
\item \textbf { Beh.:} Die offenen Mengen von $ P $ sind
2013-11-13 08:48:55 +01:00
Vereinigungen von Mengen der Form
\[ \prod _ { j \in J } U _ j \times \prod _ { i \in \mdn , i \neq j } P _ i \]
wobei $ J \subseteq \mdn $ endlich und $ U _ j \subseteq P _ j $
offen ist.
\begin { beweis}
Nach Definition der Produkttopologie bilden Mengen
der Form
2014-01-26 22:43:30 +01:00
\[ \prod _ { i \in J } U _ j \times \prod _ { \overset { i \in \mdn } { i \notin J } } P _ i, \text { wobei } J \subseteq \mdn \text { endlich und } U _ j \subseteq P _ j \text { offen } \forall { j \in J } \]
2013-11-13 08:48:55 +01:00
eine Basis der Topologie. Damit sind die offenen
Mengen von $ P $ Vereinigungen von Mengen der obigen
Form. $ \qed $
\end { beweis}
2013-11-26 23:00:58 +01:00
\item \textbf { Beh.:} Die Zusammenhangskomponenten von $ P $
2013-11-13 08:48:55 +01:00
sind alle einpunktig.\xindex { Total Unzusammenhängend}
\begin { beweis}
Es seinen $ x,y \in P $ und $ x $ sowie $ y $ liegen in der
gleichen Zusammenhangskomponente $ Z \subseteq P $ .
Da $ Z $ zusammenhängend ist und $ \forall { i \in I } : p _ i : P \rightarrow P _ i $
ist stetig, ist $ p _ i ( Z ) \subseteq P _ i $ zusammenhängend
für alle $ i \in \mdn $ . Die zusammenhängenden Mengen
von $ P _ i $ sind genau $ \Set { 0 } $ und $ \Set { 1 } $ , d.~h.
für alle $ i \in \mdn $ gilt entweder $ p _ i ( Z ) \subseteq \Set { 0 } $
oder $ p _ i ( Z ) \subseteq \Set { 1 } $ . Es sei $ z _ i \in \Set { 0 , 1 } $
so, dass $ p _ i ( Z ) \subseteq \Set { z _ i } $ für alle $ i \in \mdn $ .
Dann gilt also:
\[ \underbrace { p _ i ( x ) } _ { = x _ i } = z _ i = \underbrace { p _ i ( y ) } _ { = y _ i } \forall i \in \mdn \]
Somit folgt: $ x = y \qed $
\end { beweis}
\end { enumerate}
\end { solution}
2013-12-02 22:50:18 +01:00
2013-12-13 13:47:16 +01:00
\begin { solution} [\ref { ub3:aufg1} ]
2014-01-13 20:33:39 +01:00
\begin { enumerate} [label=(\alph * )]
\item \textbf { Beh.:} $ \GL _ n ( \mdr ) $ ist nicht kompakt.\\
\textbf { Bew.:} $ \det : \GL _ n ( \mdr ) \rightarrow \mdr \setminus \Set { 0 } $
ist stetig. Außerdem ist
$ \det ( \GL _ n ( \mdr ) ) = \mdr \setminus \Set { 0 } $ nicht
2014-01-26 22:43:30 +01:00
kompakt. $ \overset { \ref { kor: 5 . 6 } } { \Rightarrow } $
2014-01-13 20:33:39 +01:00
$ \GL _ n ( \mdr ) $ ist nicht kompakt. $ \qed $
\item \textbf { Beh.:} $ \SL _ 1 ( \mdr ) $ ist nicht kompakt, für $ n > 1 $ ist $ \SL _ n ( \mdr ) $ kompakt.\\
\textbf { Bew.:} Für $ \SL _ 1 ( \mdr ) $ gilt:
$ \SL _ 1 ( \mdr ) = \Set { A \in \mdr ^ { 1 \times 1 } | \det A = 1 } = \begin { pmatrix } 1 \end { pmatrix } \cong \Set { 1 } $ .
2014-01-26 22:43:30 +01:00
$ \overset { \ref { kor: 5 . 6 } } { \Rightarrow } \SL _ 1 ( \mdr ) $ ist
2014-01-13 20:33:39 +01:00
kompakt.\\
$ \SL _ n ( \mdr ) \subseteq \GL _ n ( \mdr ) $ lässt sich mit einer
Teilmenge des $ \mdr ^ { n ^ 2 } $ identifizieren. Nach \cref { satz:heine-borel}
sind diese genau dann kompakt, wenn sie beschränkt und
2014-02-08 21:23:01 +01:00
abgeschlossen sind. Definiere nun für für $ n \in \mdn _ { \geq 2 } , m \in \mdn $ :
\[ A _ m = \text { diag } _ n ( m, \frac { 1 } { m } , \dots , 1 ) \]
2014-01-13 20:33:39 +01:00
Dann gilt: $ \det A _ m = 1 $ , d.~h. $ A _ m \in \SL _ n ( \mdr ) $ ,
und $ A _ m $ ist unbeschränkt, da $ \| A _ m \| _ \infty = m \xrightarrow [ m \rightarrow \infty ] { } \infty $ .$ \qed $
\item \textbf { Beh.:} $ \praum ( \mdr ) $ ist kompakt.\\
\textbf { Bew.:} $ \praum ( \mdr ) \cong S ^ n / _ { x \sim - x } $ .
Per Definition der Quotiententopologie ist die Klassenabbildung stetig.
Da $ S ^ n $ als abgeschlossene und beschränkte Teilmenge
2014-01-26 22:43:30 +01:00
des $ \mdr ^ { n + 1 } $ kompakt ist $ \overset { \ref { kor: 5 . 6 } } { \Rightarrow } $
2014-01-13 20:33:39 +01:00
$ \praum ( \mdr ) $ ist kompakt. $ \qed $
\end { enumerate}
2013-12-13 13:47:16 +01:00
\end { solution}
2014-02-08 16:50:16 +01:00
\begin { solution} [\ref { ub3:meinsExtra} ]
Die Definition von Homöomorphismus kann auf \cpageref { def:homoeomorphismus}
nachgelesen werden.
2014-02-16 11:46:14 +01:00
\begin { definition} \xindex { Homomorphismus} %
2014-02-08 16:50:16 +01:00
Seien $ ( G, * ) $ und $ ( H, \circ ) $ Gruppen und
$ \varphi :G \rightarrow H $ eine Abbildung.
2014-02-16 11:46:14 +01:00
$ \varphi $ heißt \textbf { Homomorphismus} , wenn
2014-02-08 16:50:16 +01:00
\[ \forall g _ 1 , g _ 2 \in G: \varphi ( g _ 1 * g _ 2 ) = \varphi ( g _ 1 ) \circ \varphi ( g _ 2 ) \]
gilt.
\end { definition}
Es folgt direkt:
\begin { bspenum}
\item Sei $ X = \mdr $ mit der Standarttopologie und $ \varphi _ 1 : \id _ \mdr $ und $ \mdr = ( \mdr , + ) $ . Dann ist $ \varphi _ 1 $ ein Gruppenhomomorphismus und ein Homöomorphismus.
\item Sei $ G = ( \mdz , + ) $ und $ H = ( \mdz / 3 \mdz , + ) $ . Dann ist $ \varphi _ 2 : G \rightarrow H, x \mapsto x \mod 3 $ ein Gruppenhomomorphismus.
Jedoch ist $ \varphi _ 2 $ nicht injektiv, also sicher kein Homöomorphismus.
\item Sei $ X $ ein topologischer Raum. Dann ist $ \id _ X $ ein Homöomorphismus. Da keine Verknüpfung auf $ X $ definiert wurde, ist $ X $ keine Gruppe und daher auch kein Gruppenhomomorphismus.
\end { bspenum}
Also: Obwohl die Begriffe ähnlich klingen, werden sie in ganz unterschiedlichen
Kontexten verwendet.
\end { solution}
2014-02-16 11:46:14 +01:00
\begin { solution} [\ref { ub3:meinsExtra2} ]
Die Definition einer Isotopie kann auf \cpageref { def:Isotopie} nachgelesen
werden, die einer Isometrie auf \cpageref { def:Isometrie} .
\begin { definition} \xindex { Isomorphismus} %
Seien $ ( G, * ) $ und $ ( H, \circ ) $ Gruppen und
$ \varphi :G \rightarrow H $ eine Abbildung.
$ \varphi $ heißt \textbf { Isomorphismus} , wenn $ \varphi $ ein bijektiver
Homomorphismus ist.
\end { definition}
Eine Isotopie ist also für Knoten definiert, Isometrien machen nur in
metrischen Räumen Sinn und ein Isomorphismus benötigt eine Gruppenstruktur.
\end { solution}
2013-12-02 22:50:18 +01:00
\begin { solution} [\ref { ub4:aufg1} ]
\begin { enumerate} [label=(\alph * )]
\item \textbf { Vor.:} Sei $ M $ eine topologische Mannigfaltigkeit.\\
\textbf { Beh.:} $ M $ ist wegzusammehängend $ \gdw M $ ist zusammenhängend
\begin { beweis}
\enquote { $ \Rightarrow $ } : Da $ M $ insbesondere ein
topologischer Raum ist folgt diese Richtung direkt
2014-01-20 20:41:18 +01:00
aus \cref { kor:wegzusammehang-impliziert-zusammenhang} .
2013-12-02 22:50:18 +01:00
\enquote { $ \Leftarrow $ } : Seien $ x,y \in M $ und
\[ Z : = \Set { z \in M | \exists \text { Weg von } x \text { nach } z } \]
Es gilt:
\begin { enumerate} [label=(\roman * )]
\item $ Z \neq \emptyset $ , da $ M $ lokal wegzusammenhängend ist
\item $ Z $ ist offen, da $ M $ lokal wegzusammenhängend ist
\item $ Z ^ C : = \Set { \tilde { z } \in M | \nexists \text { Weg von } x \text { nach } \tilde { z } } $ ist offen
Da $ M $ eine Mannigfaltigkeit ist, existiert zu jedem
$ \tilde { z } \in Z ^ C $ eine offene und wegzusammenhängende Umgebung
$ U _ { \tilde { z } } \subseteq M $ .
Es gilt sogar $ U _ { \tilde { z } } \subseteq Z ^ C $ , denn
gäbe es ein $ U _ { \tilde { z } } \ni \overline { z } \in Z $ ,
so gäbe es Wege $ \gamma _ 2 : [ 0 , 1 ] \rightarrow M, \gamma _ 2 ( 0 ) = \overline { z } , \gamma _ 2 ( 1 ) = x $
und $ \gamma _ 1 : [ 0 , 1 ] \rightarrow M, \gamma _ 1 ( 0 ) = \tilde { z } , \gamma _ 1 ( 1 ) = \overline { z } $ .
Dann wäre aber
2014-01-30 21:53:35 +01:00
\begin { align*}
\gamma :[0,1] & \rightarrow M,\\
\gamma (x) & = \begin { cases}
\gamma _ 1(2x) & \text { falls } 0 \leq x \leq \frac { 1} { 2} \\
\gamma _ 2(2x-1) & \text { falls } \frac { 1} { 2} < x \leq 1
\end { cases}
\end { align*}
2013-12-02 22:50:18 +01:00
ein stetiger Weg von $ \tilde { z } $ nach $ x $
$ \Rightarrow $ Widerspruch.
Da $ M $ zusammenhängend ist und $ M = \underbrace { Z } _ { \mathclap { \text { offen } } } \cup \underbrace { Z ^ C } _ { \mathclap { \text { offen } } } $ ,
sowie $ Z \neq \emptyset $ folgt $ Z ^ C = \emptyset $ .
Also ist $ M = Z $ wegzusammenhängend.$ \qed $
\end { enumerate}
\end { beweis}
\item \textbf { Beh.:} $ X $ ist wegzusammenhängend.\\
\begin { beweis}
$ X: = ( \mdr \setminus \Set { 0 } ) \cup \Set { 0 _ 1 , 0 _ 2 } $
und $ ( \mdr \setminus \Set { 0 } ) \cup \Set { 0 _ 2 } $ sind
homöomorph zu $ \mdr $ . Also sind die einzigen kritischen
Punkte, die man nicht verbinden können könnte
$ 0 _ 1 $ und $ 0 _ 2 $ .
Da $ ( \mdr \setminus \Set { 0 } ) \cup \Set { 0 _ 1 } $ homöomorph
zu $ \mdr $ ist, exisitert ein Weg $ \gamma _ 1 $ von $ 0 _ 1 $
zu einem beliebigen Punkt $ a \in \mdr \setminus \Set { 0 } $ .
Da $ ( \mdr \setminus \Set { 0 } ) \cup \Set { 0 _ 2 } $ ebenfalls
homöomorph zu $ \mdr $ ist, existiert außerdem ein Weg
$ \gamma _ 2 $ von $ a $ nach $ 0 _ 2 $ . Damit existiert ein
(nicht einfacher)
Weg $ \gamma $ von $ 0 _ 1 $ nach $ 0 _ 2 $ . $ \qed $
\end { beweis}
\end { enumerate}
\end { solution}
2013-12-13 13:47:16 +01:00
2014-01-13 21:33:49 +01:00
%Das scheint mir etwas zu lang zu sein...
%\begin{solution}[\ref{ub7:aufg1}]
% \textbf{Beh.:} $H_k = \begin{cases}\mdr &\text{für } k\in \Set{0,1}\\
% 0 &\text{für } k \geq 2$
% \newcommand{\triangleSimplizialkomplex}{\mathord{\includegraphics[height=5ex]{figures/triangleSimplizialkomplex.pdf}}}
% \textbf{Bew.:} $S^1$ ist homöomorph zum Simplizialkomplex
% $X = \triangleSimplizialkomplex$, d.~h. dem Rand
% von $\Delta^2$. Es gilt:
% \[X = \Set{\underbrace{v_0, v_1, v_2}_{A_0(X)}, \underbrace{\Delta (v_1, v_2)}_{=: a_0}, \underbrace{\underbrace{\Delta (v_0, v_2)}_{=: a_1}, \underbrace{\Delta(v_0, v_1)}_{=: a_2}}_{A_1(X)}}\]
% Damit folgt:
% \begin{enumerate}
% \item Für $k \geq 2$ ist $C_k(X) \cong 0$, da es in diesen
% Dimensionen keine Simplizes gibt, d.~h. $A_k(X) = \emptyset$ gilt.\\
% Also: $H_k(X) \cong 0 \; \forall k \geq 2$
% \item $C_0(X) = \Set{\sum_{i=0}^2 c_i v_i | c_i \in \mdr}$, da
% $A_0(x)$ Basis von $C_0(X)$ ist;\\
% $C_1(X) = \Set{\sum_{i=0}^2 c_i a_i | c_i \in \mdr}$, da
% $A_1(X)$ Basis von $C_1(X)$ ist.
% \item Für die Randabbildungen $d_i: C_i(X) \rightarrow C_{i-1}(X)$ gilt:
% $d_0 \equiv 0$, $d_1: C_1(X) \rightarrow C_0(X)$ ist definiert durch
% $d_1(a_k) = \sum_{i=0}^1 (-1)^i \partial_i(a_k) = \partial_0 (a_k) - \partial_1(a_k) \; \forall k \in \Set{0,1,2}$
% \end{enumerate}
%\end{solution}
%Auch diese Aufgabe ist zu lang
%\begin{solution}[\ref{ub7:aufg3}]
%
%\end{solution}
2014-01-25 13:33:34 +01:00
2014-01-28 07:48:12 +01:00
\begin { solution} [\ref { ub11:aufg3} ]
\textbf { Vor.:} Sei $ ( X, d ) $ eine absolute Ebene, $ A, B, C \in X $
und $ \triangle ABC $ ein Dreieck.
\begin { enumerate} [label=(\alph * )]
\item \textbf { Beh.:} $ \overline { AB } \cong \overline { AC } \Rightarrow \angle ABC \cong \angle ACB $ \\
\textbf { Bew.:} Sei $ \overline { AB } \cong \overline { AC } $ .\\
$ \Rightarrow \exists $ Isometrie $ \varphi $ mit $ \varphi ( B ) = C $ und
$ \varphi ( C ) = B $ und $ \varphi ( A ) = A $ .\\
$ \Rightarrow \varphi ( \angle ABC ) = \angle ACB $ \\
$ \Rightarrow \angle ABC \cong \angle ACB \qed $
\item \textbf { Beh.:} Der längeren Seite von $ \triangle ABC $ liegt der größere Winkel gegenüber und
umgekehrt.\\
\textbf { Bew.:} Sei $ d ( A,C ) > d ( A,B ) $ . Nach \ref { axiom:3.1}
gibt es $ C' \in AC ^ + $ mit $ d ( A, C' ) = d ( A,B ) $ \\
$ \Rightarrow C' $ liegt zwischen $ A $ und $ C $ .\\
Es gilt $ \measuredangle ABC' < \measuredangle ABC $ und
aus \cref { ub11:aufg3.a} folgt: $ \measuredangle ABC' = \measuredangle AC' B $ .\\
$ \angle BC' A $ ist ein nicht anliegender Außenwinkel zu
$ \angle BCA \xRightarrow { \crefabbr { bem: 14 . 9 } } \measuredangle BC' A > \measuredangle BCA $ \\
$ \Rightarrow \measuredangle BCA < \measuredangle BC' A = \measuredangle ABC' < \measuredangle ABC $
Sei umgekehrt $ \measuredangle ABC > \measuredangle BCA $ ,
kann wegen 1. Teil von \cref { ub11:aufg3.b} nicht
$ d ( A,B ) > d ( A,C ) $ gelten.\\
Wegen \cref { ub11:aufg3.a} kann nicht $ d ( A,B ) = d ( A,C ) $
gelten.\\
$ \Rightarrow d ( A,B ) < d ( A, C ) \qed $
\item \textbf { Vor.:} Sei $ g $ eine Gerade, $ P \in X $ und $ P \notin g $ \\
\textbf { Beh.:} $ \exists ! $ Lot\\
\textbf { Bew.:} ÜB10 A4(a): Es gibt Geradenspiegelung $ \varphi $
an $ g $ . $ \varphi $ vertauscht die beiden Halbebenen bzgl.
$ g $ .\\
$ \Rightarrow \varphi ( P ) P $ schneidet $ g $ in $ F $ .
2014-02-09 16:32:39 +01:00
%Nach ÜB 10 A4(a):
Es gibt eine Geradenspiegelung $ \varphi $ an $ g $ .
$ \varphi $ vertauscht die beiden Halbebenen bzgl. $ g $
$ \Rightarrow \varphi ( P ) P $ schneidet $ g $ in $ F $ .
Sei $ A \in g \setminus \Set { F } $ . Dann gilt $ \varphi ( \angle AFP ) = \angle AF \varphi ( P ) = \pi $
$ \Rightarrow \angle AFP $ ist rechter Winkel.
Gäbe es nun $ G \in g \setminus \Set { F } $ , so dass $ PG $ weiteres Lot von $ P $ auf $ g $ ist,
wäre $ \triangle PFG $ ein Dreieck mit zwei rechten Innenwinkeln (vgl. \cref { fig:two-perpendiculars} ).
\begin { figure} [htp]
\centering
\input { figures/two-perpendiculars.tex}
\caption { Zwei Lote zu einer Geraden $ g $ durch einen Punkt $ P $ }
\label { fig:two-perpendiculars}
\end { figure}
Nach \cref { folgerung:14.10} ist die Summe von zwei Innenwinkeln immer $ < \pi $ \\
$ \Rightarrow G $ gibt es nicht. $ \qed $
2014-01-28 07:48:12 +01:00
\end { enumerate}
\end { solution}
2014-01-25 13:33:34 +01:00
\begin { solution} [\ref { ub-tut-24:a1} ]
Sei $ f \parallel h $ und \obda $ f \parallel g $ .
$ f \nparallel h \Rightarrow f \cap h \neq \emptyset $ , sei also $ x \in f \cap h $ .
Mit Axiom \ref { axiom:5} folgt: Es gibt höchstens eine Parallele
zu $ g $ durch $ x $ , da $ x \notin g $ . Diese ist $ f $ , da $ x \in f $
und $ f \parallel g $ . Da aber $ x \in h $ , kann $ h $ nicht parallel
zu $ g $ sein, denn ansonsten gäbe es zwei Parallelen zu $ g $ durch
$ x $ ($ f \neq h $ ).
$ \Rightarrow g \nparallel h $ $ \qed $
\end { solution}
2014-02-20 14:38:02 +01:00
\begin { solution} [\ref { ub-tut-24:a3} ]\xindex { Kongruenzsatz!SSS} %
Sei $ ( X,d,G ) $ eine Geometrie, die \ref { axiom:1} -\ref { axiom:4} erfüllt.
Seien außerdem $ \triangle ABC $ und $ \triangle A'B' C' $ Dreiecke, für die gilt:
2014-01-25 13:33:34 +01:00
\begin { align*}
d(A, B) & = d(A', B')\\
2014-02-20 14:38:02 +01:00
d(A, C) & = d(A', C')\\
d(B, C) & = d(B', C')
2014-01-25 13:33:34 +01:00
\end { align*}
2014-02-20 14:38:02 +01:00
Sei $ \varphi $ die Isometrie mit $ \varphi ( A ) = A' $ , $ \varphi ( B ) = B' $ und
$ \varphi ( C' ) $ liegt in der selben Halbebene bzgl. $ AB $ wie $ C $ . Diese
Isometrie existiert wegen \ref { axiom:4} .
2014-01-25 13:33:34 +01:00
2014-02-20 14:38:02 +01:00
Es gilt $ d ( A,C ) = d ( A', C' ) = d ( \varphi ( A' ) , \varphi ( C' ) ) = d ( A, \varphi ( C' ) ) $
und $ d ( B,C ) = d ( B', C' ) = d ( \varphi ( B' ) , \varphi ( C' ) ) = d ( B, \varphi ( C' ) ) $ .\\
$ \xRightarrow { \crefabbr { kor: 14 . 6 } } C = \varphi ( C ) $ .
Es gilt also $ \varphi ( \triangle A'B'C' ) = \triangle ABC $ . $ \qed $
2014-02-09 16:32:39 +01:00
\end { solution}